• 検索結果がありません。

An explicit upper bound of the argument of Dirichlet $L$-functions on the generalized Riemann hypothesis (Analytic Number Theory : Distribution and Approximation of Arithmetic Objects)

N/A
N/A
Protected

Academic year: 2021

シェア "An explicit upper bound of the argument of Dirichlet $L$-functions on the generalized Riemann hypothesis (Analytic Number Theory : Distribution and Approximation of Arithmetic Objects)"

Copied!
6
0
0

読み込み中.... (全文を見る)

全文

(1)94. 数理解析研究所講究録 第2013巻 2016年 94-99. An. explicit. upper bound of the on. the. generalized. argument of Dirichlet L‐functions. hypothesis. Riemann. 八戸工業高等専門学校総合科学教育科若狭尊裕 Takahiro Wakasa. Hachinohe National. College. of. Technology. Abstract. by the argument of Dirichlet An explicit upper bound of the function S(t) defined by the argument of the Riemann zeta‐function, have been obtained by A. Fujii [1]. Our result is obtained by applying the idea of Fujii’s result on S(t) The constant part of the explicit upper bound of S(t, $\chi$) in this paper does not depend on $\chi$ Our proof does not covor the case q=1 and indeed gives a better bound than the one of Fujii that covers the case q=1. We prove an explicit upper bound of the function S(t, $\chi$) defined L ‐functions attached to a primitive Dirichlet character $\chi$(\mathrm{m}\mathrm{o}\mathrm{d} q>1) ,. .. ,. .. .. Introduction. 1. The argument of the Riemann zeta‐function. on. the clitical line defined. by. S(t)=\displaystyle \frac{1}{ $\pi$}\arg $\zeta$(\frac{1}{2}+it) when t is not the ordinate of. lines of. a zero. connecting 2, 2+it and ,. $\zeta$(s). ,. we. define. \displaystyle \frac{1}{2}+it. of ,. $\zeta$(s). .. This is obtained. starting with the value. continuous variation. by. zero.. Also,. along. the. straight. when t is the ordinate of. a zero. S(t)=\displaystyle \frac{1}{2}\{S(t+0)+S(t-0 consider the argument of Dirichlet L ‐functions. Let L(s, $\chi$) be the Dirichlet L ‐fUnction, where Now, s= $\sigma$+it is a cotnplex variable, assosiated with a primitive Dirichlet character $\chi$(\mathrm{m}\mathrm{o}\mathrm{d} q>1) Here, we we. .. denote the non‐trivial. Then,. L(s, $\chi$) by $\rho$( $\chi$)= $\beta$( $\chi$)+i $\gamma$( $\chi$) ordinate of a zero of L(s, $\chi$) we define. zeros. when t is not the. of. where. $\beta$( $\chi$). and. $\gamma$( $\chi$). are. real numbers.. ,. S(t $\chi$)=\displaystyle \frac{1}{ $\pi$}\arg L(\frac{1}{2}+it $\chi$) This is. given by. with the value. continuous variation. zero.. Also, when. along. the. straight. t is the ordinate of. line s= $\sigma$+it ,. a zero. of. L(s, $\chi$). ,. as $\sigma$ we. varies from +\infty to. define. S(t $\chi$)=\displaystyle \frac{1}{2}\{S(t+0 $\chi$)+S(t-0_{\rangle} $\chi$ Selberg proved. S(t, $\chi$)=O(\log q(t+1)) and under the. generalized. Riemann. hypothesis (GRH). S(t, $\chi$)=O(\displaystyle \frac{\log q(t+1)}{\log\log q(t+3)}) in. Selberg [2].. The purpose of the present article is to prove the. following. result.. \displayte\frac{1}2. ,. starting.

(2) 95. Theorem 1.. A_{\mathrm{b}6}\cdot uming. GRH.. Then, for q>1. |S(t, $\chi$)|<0.804\displaystyle \cdot\frac{\log q(t+1)}{\log\log q(t+3)}+O(\frac{\log q(t+3)}{(\log\log q(t+3))^{2} ) The constant 0.804. .. obviously does not depend on $\chi$ And we don’t know anything concerning the implied constant of the error term does not depend on q Our result does not include the case of the function S(t) since we assume q>1 An explicit upper bound of the function S(t) is obtained by A. Fujii [1], where the value is 0.83. The basic policy of the proof of this theorem is based on A. Fujii [1]. In the proof, S(t, $\chi$) is seperated by three parts M_{1}, M_{2} and M_{3} Fujii’s idea of [1] is applied to all parts. But we need Lemma 1, which is an explicit formula for \displaystyle \frac{L'}{L}(s, $\chi$) This lemma is an analogue of Selberg’s result. optimality. Also,. .. the. .. .. .. .. Some notations and. 2 Here. we. introduce the. Let s= $\sigma$+it. Also,. we. .. following. a. lemma. notations.. $\sigma$\displaystyle \geq\frac{1}{2}. We suppose that. put. and t\geq 2. .. Let. x. be. a. positive number satisfying 4\leq x\leq t^{2}.. $\sigma$_{1}=\displaystyle \frac{1}{2}+\frac{1}{\log x} and. $\Lambda$_{x}(n)=\left\{ begin{ar y}{l $\Lambda$(n)&\mathrm{f}\mathrm{o}\mathrm{}1\leqn\leqx,\ $\Lambda$(n)\frac{\log\frac{x^2}{n}\logx}&\mathrm{f}\mathrm{o}\mathrm{}x\leqn\leqx^{2}, \end{ar y}\right. with. $\Lambda$(n)=\left\{ begin{ar y}{l \logp&\mathrm{i}\mathrm{f}n=p^{k}\mathrm{w}\mathrm{i}\mathrm{t}\mathrm{h}\mathrm{a}\mathrm{p}\mathrm{}\mathrm{i}\mathrm{ }\mathrm{e}p\mathrm{a}\mathrm{n}\mathrm{d}\mathrm{a}\mathrm{n}\mathrm{i}\mathrm{n}\mathrm{t}\mathrm{e}\mathrm{g}\mathrm{e}\mathrm{}k\geq1,\ 0&\mathrm{o}\mathrm{t}\mathrm{h}\mathrm{e}\mathrm{i}\mathrm{w}\mathrm{i}\mathrm{s}\mathrm{e}. \end{ar y}\right. Using. these. notations,. we. prove the. following. lemma.. Lemma 1. Assume the GRH. Let t\geq 2 and x>0 such that there exist. and $\omega$' such that. $\omega$. | $\omega$|\leq 1 and-1\leq$\omega$'\leq 1. ,. we. 4\leq x\leq t^{2}. .. Then. for. have. $\sigma$\displaystyle \geq$\sigma$_{1}=\frac{1}{2}+\frac{1}{\log x},. \displayst le\frac{L'} ($\sigma$+it, $\chi$)=-\sum_{n<x^{2}\frac{$\Lambda$_{x'}.(n)}{n^ $\sigma$+it} $\chi$(n)-\frac{x^\frac{1}2-$\sigma$}(1+x^{\frac{1}2-$\sigma$})$\omega$}{1-\frac{1}e(1+\frac{1}e)$\omega$}\mathfrak{R}(\sum_{n<x^{2}\frac{$\Lambda$_{x}(n)}{n^$\sigma$_{1}+tl}$\chi$(n) +\displayst le\frac{x^\frac{1}2-$\sigma$}(1+x^{\frac{1}2-$\sigma$})$\omega$}{1-\frac{1}e(1+\frac{1}e)$\omega$'}\cdot\frac{1}2\logq(t+1)+O(x^{\frac{1}2-$\sigma$}). This is. an. analogue. Lemma 2. Let a=0 0,. 1, 2,. ). and. of Lcmma 2 of A.. if $\chi$(-1)=1. s\neq $\rho$( $\chi$). ,. we. ,. .. Fujii [1].. and a=1. if $\chi$(-1)=-1. .. Then, for x>1, s\neq-2q-a. have. \displaystyle \frac{L'}{L}(s, $\chi$)=-\sum_{n<x^{2} \frac{$\Lambda$_{x}(n)}{n^{s} $\chi$(n)+\frac{1}{\log x}\sum_{q=0}^{\infty}\frac{x^{-2q-a s}-x^{-2(2\mathrm{q}+a+s)} {(2q+a+s)^{2} +\displaystyle\frac{1}{\logx}\sum_{$\rho$}\frac{x^{$\rho$-s}x^{2($\rho$-s)}{(s-$\rho$)^{2}.. (q=.

(3) 96. Lemma 2 is similar to Lemma 15 of. If. a=\displaystyle \max(1, $\sigma$). we. ,. Selberg [2].. We write here. only. a. sketch of the. proof of Lemma. 2.. have. \displaystyle\sum_{n<x^{2} \frac{$\Lambda$_{x}(n)}{n^{s} $\chi$(n)=\frac{\mathrm{l} {2$\pi$i\logx}\int_{a-\inftyi}^{a+\inftyi}\frac{x^{z-s}-x^{2(z-s)} {(z-s)^{2} \cdot\frac{L'}{L}(z, $\chi$)dz. We consider residues which. point. z=\mathrm{s} ,. the residue is. \displaystyle \frac{x^{-2q-a-s}-x^{-2(2q+a+s)} {(2\mathrm{q}+a+s)^{2}. Lemma2.. we. -(\displaystyle \log x)\frac{L'}{L}(s, $\chi$). At the. .. Proof of Lemma. encounter when .. s= $\rho$ of. zeros. we. move. At the. L(s, $\chi$). 1. Assume the GRH. In. zeros. ,. path of integration to the left. At the -2q-a (q=0,1,2, \cdots) the residues are. the. the residues. ,. are. Lemma2, since for. \displaystyle\frac{x^{$\rho$-s}x^{2($\rho$-\mathrm{s}) {(s-$\rho$)^{2}. Thus,. .. we. obtain. $\sigma$\displaystyle \geq$\sigma$_{1}=\frac{1}{2}+\frac{1}{\log x}. |\displaystyle\frac{1}{\logx}\sum_{$\rho$}\frac{x^{$\rho$-s}x^{2($\rho$-s)}{(s-$\rho$)^{2}|\leqx^{\frac{1}{2}-$\sigma$}(1+x^{\frac{1}{2}$\sigma$})\sum_{$\gam a$}\frac{$\sigma$_{1}-\frac{1}{2}{($\sigma$_{1}-\frac{1}{2})^{2}+(t-$\gam a$)^{2}, we. have. \displaystyle\frac{1}{\logx}\sum_{$\rho$}\frac{x^{$\rho$-s}x^{2($\rho$-s)}{(s-$\rho$)^{2}=x^{\frac{1}{2}-$\sigma$}(1+x^{\frac{1}{2}-$\sigma$})$\omega$\sum_{$\gam a$}\frac{$\sigma$_{1}-\frac{1}{2}{($\sigma$_{1}-\frac{1}{2})^{2}+(t-$\gam a$)^{2}, where. | $\omega$|\leq 1. .. Hence. by. Lemma. 2,. we. have for $\sigma$\geq$\sigma$_{1}. \displaystyle\frac{L'}{L}($\sigma$+it, $\chi$)=-\sum_{n<x^{2}\frac{$\Lambda$_{x}(n)}{n^{$\sigma$+$\iota$t} $\chi$(n)+O(\frac{x^{\frac{\mathrm{l}{2}-$\sigma$}{\logx}). +x^{\frac{1}{2}-$\sigma$}(1+x^{\frac{1}{2}-$\sigma$})$\omega$\displaystyle\sum_{$\gam a$}\frac{$\sigma$_{1}-\frac{1}{2}{($\sigma$_{1}-\frac{1}{2})^{2}+(t-$\gam a$)^{2}. In. particular,. since. x^{\frac{1}{2}- $\sigma$}\displaystyle \leq x^{-\frac{1}{\log x} =\frac{1}{e}. for $\sigma$\geq$\sigma$_{1} ,. we. (1). .. get. \displaystyle\mathfrak{R}\frac{L'}{L}($\sigma$_{1}+it, $\chi$)=-\mathfrak{R}(\sum_{n<x^{2} \frac{$\Lambda$_{x}(n)}{n^{$\sigma$_{1}+it} $\chi$(n) +O(\frac{1}{\logx}). +\displaystyle\frac{1}{e}(1+\frac{1}{e})$\omega$'\sum_{$\gam a$}\frac{$\sigma$_{1}-\frac{1}{2}{($\sigma$_{1}-\frac{1}{2})^{2}+(t-$\gam a$)^{2}. -1\leq$\omega$'\leq 1. Here, since by p.. (2). ,. where. 46 of. Selberg [2]. \displaystyle \mathfrak{R}\frac{L'}{L}(s, $\chi$)=\mathfrak{R}(-\frac{1}{2}\log\frac{q}{ $\pi$}-\frac{1}{2}\log(\frac{s+a}{2}) +\sum_{ $\gam a$}\frac{ $\sigma$-\frac{1}{2} {( $\sigma$-\frac{1}{2})^{2}+(t- $\gam a$)^{2} +O(1) we. ,. get for t\geq 2. \displaystyle \mathfrak{R}\frac{L'}{L}($\sigma$_{1}+it, $\chi$)=-\frac{1}{2}\log q(t+1)+\sum_{ $\gam a$}\frac{$\sigma$_{1}-\frac{1}{2} {($\sigma$_{1}-\frac{1}{2})^{2}+(t- $\gam a$)^{2} +O(1) By (2). and. (3). we. (3). .. have. (1-\displaystyle\frac{1}{e}(1+\frac{1}{e})$\omega$')\sum_{$\gam a$}\frac{$\sigma$_{1}-\frac{1}{2} {($\sigma$_{1}-\frac{1}{2})^{2}+(t-$\gam a$)^{2} =-\displaystyle \mathfrak{R}(\sum_{n<x^{2} \frac{$\Lambda$_{x}(n)}{n^{$\sigma$_{1}+ $\iota$ t} $\chi$(n) +\frac{1}{2}\log q(t+1)+O(\frac{1}{\log x})+O(1) Inserting the above inequality. to. (1),. we. .. obtain Lemma 1. 口.

(4) 97. Proof of Theorem 1. 3 The. quantity S(t, $\chi$). is. separated. into the. following. three parts.. S(t, $\chi$)=-\displaystyle \frac{1}{ $\pi$}\{\Im\int_{$\sigma$_{1} ^{\infty}\frac{L'}{L}( $\sigma$+it, $\chi$)d $\sigma$+\Im\{($\sigma$_{1}-\frac{1}{2})\frac{L'}{L}($\sigma$_{1}+it, $\chi$)\} -\displaystyle\Im\int_{\frac{1}{2} ^{$\sigma$_{1} \{ frac{L'}{L}($\sigma$_{1}+it, $\chi$)-\frac{L'}{L}($\sigma$+it, $\chi$)\}d$\sigma$\} =-\displaystyle \frac{1}{ $\pi$}\Im(M_{1}+M_{2}+M_{3}). ,. say.. First,. we. estimate. M_{1}. .. By. Lemma 1. we. have. M_{1}=\displayst le\int_{$\sigma$_{1}^{\infty}\{- sum_{n<x^{2}\frac{$\Lambda$_{n}(x){n^ $\sigma$+it} $\chi$(n)-\frac{x^\frac{1}2-$\sigma$}(1+x^{\frac{1}2-$\sigma$}) \omega$}{1-\frac{1}\mathrm{e}(1+\frac{1}\mathrm{e})$\omega$}\mathfrak{R}(\sum_{n<x^{2}\frac{$\Lambda$_{n}(x){n^$\sigma$_{1}+it}$\chi$(n) +\displaystyle\frac{x^{\frac{1}2-$\sigma$}(1+x^{\frac{1}2-$\sigma$})$\omega$}{1-\frac{1}e(1+\frac{1}e)$\omega$}\cdot\frac{1}2\logq(t+1)+O(x^{\frac{1}2-$\sigma$})\d$\sigma$ =-\displaystyle \sum_{n<x^{2} \frac{$\Lambda$_{n}(x)}{n^{$\sigma$_{1}+it}\log n}$\chi$(n)+$\eta$_{1}(t). say.. (4). ,. Here,. |$\eta$_{1}(t)|\displaystyle\leq\frac{1}{1-\frac{1}{e}(1+\frac{1}{e}) |\mathfrak{R}(\sum_{n<x^{2} \frac{$\Lambda$_{n}(x)}{n^{$\sigma$_{1}+it} $\chi$(n) -\frac{1}{2}\logq(t+1)| \displaystyle\times\int_{$\sigma$_{1} ^{\infty}x^{\frac{1}{2}-$\sigma$}(1+x^{\frac{1}{2}-$\sigma$})d$\sigma$+O(\int_{$\sigma$_{1} ^{\infty}x^{\frac{1}{2}-$\sigma$}d$\sigma$). \displaystyle\leq\frac{(\frac{1}{e}+\frac{1}{2e^{2}){1-\frac{1}{e}(1+\frac{1}{e})\cdot\frac{1}{2}\cdot\frac{\logq(t+1)}{\logx}+O(\frac{1}{\logx}|\sum_{n<x^{2}\frac{$\Lambda$_{n}(x)}{n^{$\sigma$_{1}+xt} $\chi$(n)|. ,. (5). say.. Next, applying. Lemma 1 to. M_{2}. ,. we. get. |M_{2}|\displaystyle\leq\frac{(\frac{1}{e}+\frac{1}{e^{2} ) {1-\frac{1}{e}(1+\frac{1}{e}) \cdot\frac{1}{2}\cdot\frac{\logq(t+1)}{\logx}+O(\frac{1}{\logx}|\sum_{n<x^{2} \frac{$\Lambda$_{n}(x)}{n^{$\sigma$_{1}+it} $\chi$(n)|. ,. say.. Next. we. estimate. M_{3} By Lemma .. 16 of. Selberg [2]. we. get. |\displayst le\Im(M_{3})|\leq|\int_{\frac{1}2}^{$\sigma$_{1}\sum_{$\gam a$}\frac{(t-$\gam a$)\{($\sigma$-\frac{1}2)^{2}-($\sigma$_{1}-\frac{1}2)^{2}\ {\($\sigma$_{1}-\frac{1}2)^{2}+(t-$\gam a$)^{2}\ {($\sigma$-\frac{1}2)^{2}+(t-$\gam a$)^{2}\ d$\sigma$|+O(\frac{1}\logx}) <\displaystyle\int_{\frac{1}{2} ^{$\sigma$_{1} \sum_{$\gam a$}N($\gam a$, $\sigma$)d$\sigma$+O(\frac{1}{\logx}). say.. Here,. we. put. \displaystyle \mathfrak{R}=\int_{\frac{1}{2} ^{$\sigma$_{1} \sum_{ $\gamma$}N( $\gamma$, $\sigma$)d $\sigma$. .. Then,. we. have. \displayst le\mathfrak{R}<\sum_{$\gam a$}\frac{($\sigma$_{1}-\frac{1}2)^{2}{($\sigma$_{1}-\frac{1}2)^{2}+(t-$\gam a$)^{2}\int_{\overline{2}^{\infty}\frac{|t-$\gam a$|}{($\sigma$-\frac{1}2)^{2}+(t-$\gam a$)^{2}d$\sigma$ \displaystyle\leq\frac{$\pi$}{2\logx}\sum_{$\gam a$}\frac{$\sigma$_{1}-\frac{1}2}{($\sigma$_{1}-\frac{1}2)^{2}+(t-$\gam a$)^{2}. (6).

(5) 98. since $\sigma$<$\sigma$_{1} for. Here, by (2). M_{3}. and. (3). we. get. \displaystyle\sum_{$\gam a$}\frac{$\sigma$_{1}-\frac{1}{2} {($\sigma$_{1}-\frac{1}{2})^{2}+(t-$\gam a$)^{2} =\frac{1}{1-\frac{1}{e}(1+\frac{1}{e})$\omega$'}\cdot\frac{1}{2}\logq(t+1). +O(|\displaystyle \sum_{n<x^{2} \frac{$\Lambda$_{n}(x)}{n^{$\sigma$_{1}+it} $\chi$(n)| +O(\frac{1}{(\log x)^{2} ). So,. .. \displaystyle \mathfrak{R}=\frac{ $\pi$}{4}\cdot\frac{1}{1-\frac{1}{\mathrm{e} (1+\frac{1}{\mathrm{e} )$\omega$'}\cdot\frac{1}{\log x}\cdot\log q(t+1) Hence. we. +O(\displaystyle \frac{1}{\log x}|\sum_{n<x^{2} \frac{$\Lambda$_{n}(x)}{n^{$\sigma$_{1}+xt} $\chi$(n)| +O(\frac{1}{(\log x)^{3} ). have. .. |\displaystyle \Im(M_{3})|\leq\frac{ $\pi$}{4}\cdot\frac{1}{1-\frac{1}{e}(1+\frac{1}{e})$\omega$'}\cdot\frac{1}{\log x}\cdot\log q(t+1). +O(\displaystyle\frac{1}{\logx}|\sum_{n<x^{2} \frac{$\Lambda$_{n}(x)}{n^{$\sigma$_{1}+$\iota$t} $\chi$(n)| +O(\frac{1}{\logx}) =$\eta$_{4}(t)+O(\displaystyle \frac{1}{\log x}|\sum_{n<x^{2} \frac{$\Lambda$_{n}(x)}{n^{$\sigma$_{1}+it} $\chi$(n)| +O(\frac{1}{\log x}). say.. Finally,. we. estimate the. sums on. right‐hand. sides of. (4), (5), (6). and. (7). By. definition of. have. |\displayst le\sum_{n<x^{2}\frac{$\Lambda$_{x}(n)}{n^{$\sigma$_{1}+$\iota$t} $\chi$(n)|\leq\sum_{n<x}\frac{$\Lambda$(n)}{n^{\frac{1}2}+\sum_{x\leqn\leqx^{2}\frac{$\Lambda$(n)\log\frac{x^2}{n}{n^{\frac{\mathrm{l} 2} \cdot\frac{1}\logx}\l frac{x}\logx}.. Similarly,. So,. |\displaystyle\sum_{n<x^{2}\frac{$\Lambda$_{x}(n)}{n^{$\sigma$_{1}+it}\logn}$\chi$(n)|\l\frac{x}{(\logx)^{2}. we see. |M_{1}|\displaystyle \leq\frac{(\frac{1}{e}+\frac{1}{2e^{2} )}{1-\frac{1}{P}(1+\frac{1}{e,}) \cdot\frac{1}{2}\cdot\frac{\log q(t+1)}{\log x}+O(\frac{x}{(\log x)^{2} ) |M_{2}|\displaystyle \leq\frac{(\frac{1}{e}.+\frac{1}{e^{2} )}{1-\frac{1}{e}(1+\frac{1}{P}) \cdot\frac{1}{2}\cdot\frac{\log q(t+1)}{\log x}+O(\frac{x}{(\log x)^{2} ). and. |M_{3}|\displaystyle \leq$\eta$_{4}(t)+O(\frac{x}{(\log x)^{2} ) For. $\eta$_{1}(t) $\eta$_{2}(t) $\eta$_{3}(t) ,. ,. and. $\eta$_{4}(t) taking ,. ,. ,. .. x=\log q(t+3)\sqrt{\log q(t+3)}. we. have. |S(t, $\chi$)|<\displaystyle\frac{1}{$\pi$}\cdot\frac{1}{1-\frac{1}{e}(1+\frac{1}{e}) \{ frac{(\frac{1}{e}+\frac{1}{2e^{2} )}{2}+\frac{(\frac{1}{\mathrm{e} +\frac{1}{e^{2} )}{2}+\frac{$\pi$}{4}\ frac{\logq(t+1)}{\logx} +O(\displaystyle \frac{x}{(\log x)^{2} ) \displaystyle \frac{\log q(t+1)}{\log\log q(t+3)}+O(\frac{\log q(t+3)}{(\log\log q(t+3))^{2} ). =0.803986. .. .. .. .. (7). ,. $\Lambda$_{x}(n). we.

(6) 99. Therefore. we. 口. obtain the theorem.. References [1]. A.. [2]. A.. Fujii, An explicit estimate in the theory of the distribution of the function, Comment. Math. Univ. Sancti Pauli, 53, (2004), 85‐114.. Selberg,. (1946),. [3]. A.. [4]. E. C.. Contributions to the. theory. of Dirichlet’s. L‐‐function, Avh.. zeros. of the Riemann zeta. Norske Vir. Akad. Oslo. \mathrm{I}:1,. No. 3, 1‐62.. Selberg,. Collected. Titchmarsh,. Works,. The. \mathrm{v}\mathrm{o}\mathrm{l}\mathrm{I} ,. 1989, Springer.. theory of the Riemann zeta‐function, Second Edition; Revised by Oxford, 1986.. Brown. Clarendon Press. D. R. Heath‐.

(7)

参照

関連したドキュメント

Our result im- proves the upper bound on the number of BSDR’s with minimal weight stated by Grabner and Heuberger in On the number of optimal base 2 representations,

“rough” kernels. For further details, we refer the reader to [21]. Here we note one particular application.. Here we consider two important results: the multiplier theorems

Splitting homotopies : Another View of the Lyubeznik Resolution There are systematic ways to find smaller resolutions of a given resolution which are actually subresolutions.. This is

Classical definitions of locally complete intersection (l.c.i.) homomor- phisms of commutative rings are limited to maps that are essentially of finite type, or flat.. The

Condition (1.2) and especially the monotonicity property of K suggest that both the above steady-state problems are equivalent with respect to the existence and to the multiplicity

Yin, “Global existence and blow-up phenomena for an integrable two-component Camassa-Holm shallow water system,” Journal of Differential Equations, vol.. Yin, “Global weak

[2])) and will not be repeated here. As had been mentioned there, the only feasible way in which the problem of a system of charged particles and, in particular, of ionic solutions

We study the classical invariant theory of the B´ ezoutiant R(A, B) of a pair of binary forms A, B.. We also describe a ‘generic reduc- tion formula’ which recovers B from R(A, B)